Bytelearn - cat image with glassesAI tutor

Welcome to Bytelearn!

Let’s check out your problem:

Let 
g be a continuous function on the closed interval 
[-1,3], where 
g(-1)=-2 and 
g(3)=-5.
Which of the following is guaranteed by the Intermediate Value Theorem?
Choose 1 answer:
(A) 
g(c)=-3 for at least one 
c between -1 and 3
(B) 
g(c)=-3 for at least one 
c between -5 and -2
(c) 
g(c)=0 for at least one 
c between -5 and -2
(D) 
g(c)=0 for at least one 
c between -1 and 3

Let g g be a continuous function on the closed interval [1,3] [-1,3] , where g(1)=2 g(-1)=-2 and g(3)=5 g(3)=-5 .\newlineWhich of the following is guaranteed by the Intermediate Value Theorem?\newlineChoose 11 answer:\newline(A) g(c)=3 g(c)=-3 for at least one c c between 1-1 and 33\newline(B) g(c)=3 g(c)=-3 for at least one c c between 5-5 and 2-2\newline(C) g(c)=0 g(c)=0 for at least one c c between 5-5 and 2-2\newline(D) g(c)=0 g(c)=0 for at least one c c between 1-1 and 33

Full solution

Q. Let g g be a continuous function on the closed interval [1,3] [-1,3] , where g(1)=2 g(-1)=-2 and g(3)=5 g(3)=-5 .\newlineWhich of the following is guaranteed by the Intermediate Value Theorem?\newlineChoose 11 answer:\newline(A) g(c)=3 g(c)=-3 for at least one c c between 1-1 and 33\newline(B) g(c)=3 g(c)=-3 for at least one c c between 5-5 and 2-2\newline(C) g(c)=0 g(c)=0 for at least one c c between 5-5 and 2-2\newline(D) g(c)=0 g(c)=0 for at least one c c between 1-1 and 33
  1. The Intermediate Value Theorem: The Intermediate Value Theorem states that if a function ff is continuous on a closed interval [a,b][a, b] and NN is any number between f(a)f(a) and f(b)f(b), then there exists at least one number cc in the interval (a,b)(a, b) such that f(c)=Nf(c) = N.
  2. Given Function and Interval: We are given that gg is continuous on the closed interval [1,3][-1,3], g(1)=2g(-1)=-2, and g(3)=5g(3)=-5. We need to find if there is a value cc in the interval [1,3][-1,3] such that g(c)g(c) equals a certain value.
  3. Option (A) Explanation: Option (A) suggests that g(c)=3g(c)=-3 for at least one cc between 1-1 and 33. Since 3-3 is between g(1)=2g(-1)=-2 and g(3)=5g(3)=-5, by the Intermediate Value Theorem, there must be at least one cc in the interval (1,3)(-1, 3) such that g(c)=3g(c)=-3.
  4. Option (B) Explanation: Option (B) is incorrect because it refers to a range of cc values between 5-5 and 2-2, which are yy-values, not xx-values. The Intermediate Value Theorem applies to xx-values within the interval [1,3][-1,3].
  5. Option (C) Explanation: Option (C) suggests that g(c)=0g(c)=0 for at least one cc between 5-5 and 2-2, which is again referring to yy-values, not xx-values. This is not what the Intermediate Value Theorem guarantees.
  6. Option (D) Explanation: Option (D) suggests that g(c)=0g(c)=0 for at least one cc between 1-1 and 33. However, since 00 is not between g(1)=2g(-1)=-2 and g(3)=5g(3)=-5, the Intermediate Value Theorem does not guarantee that there is a cc such that g(c)=0g(c)=0 in the interval [1,3][-1,3].

More problems from Domain and range of quadratic functions: equations